Thứ tự tôpô tích cực, mất 3


20

Giả sử ta có một ma trận n bởi n. Có thể sắp xếp lại các hàng và cột của nó để chúng ta có được một ma trận tam giác trên không?

Câu hỏi này được thúc đẩy bởi vấn đề này: Thứ tự tôpô tích cực

Vấn đề quyết định ban đầu ít nhất cũng khó như vấn đề này, do đó, kết quả hoàn thành NP cũng sẽ giải quyết vấn đề đó.

Chỉnh sửa: Laszlo Vegh và Andras Frank kêu gọi sự chú ý của tôi đến một vấn đề tương đương được hỏi bởi Gunter Rote: http://lemon.cs.elte.hu/egres/open/Graphs_extendable_to_a_uniquely_matchable_bipartite_graph

Chỉnh sửa: Việc giảm bớt vấn đề ban đầu như sau. Giả sử rằng DAG chỉ có hai cấp độ, những mức này sẽ tương ứng với các hàng và cột của ma trận. Ngoài ra, chúng tôi có một nút duy nhất có trọng số +1. Mọi người khác ở cấp thấp hơn có trọng số -1 và ở cấp trên +1.


Làm thế nào để bạn giảm điều này đến vấn đề ban đầu? Nhân tiện, vấn đề này có vẻ thú vị trong chính nó.
Tsuyoshi Ito

Bạn đang tìm kiếm một hoán vị để áp dụng cho cả hàng và cột hoặc hai hoán vị riêng biệt? Tôi đoán hai, vì chỉ có một vấn đề có vẻ tương đương với loại cấu trúc liên kết.
Warren Schudy

Nghĩ về nó như một biểu đồ lưỡng cực (như trong liên kết elte), họ đưa ra một điều kiện cần thiết là nó không có sơ đồ con được tạo ra từ các bản sao của K2, C4, C6, C8, v.v. các bộ phận bị chi phối bởi (1, 2, 3, ..., n) --- Tôi nghĩ rằng điều này mạnh hơn các điều kiện dựa trên cụm khác trong liên kết.
daveagp

Câu trả lời:


12

Vấn đề hóa ra là NP-đầy đủ. Bạn có thể đọc thêm chi tiết ở đâyở đây . Tóm tắt ngắn gọn:

Việc giảm là từ một vấn đề đã được hiển thị là NP-hoàn thành bởi Dasgupta, Jiang, Kannan, Li và Sweedyk: đưa ra một biểu đồ lưỡng cực G và một số nguyên k, quyết định xem G có biểu đồ con cảm ứng trên các nút 2k có thể được mở rộng tới được kết hợp độc đáo. Stéphane Vialette đã quan sát thấy rằng điều này làm giảm phiên bản phù hợp duy nhất của lưỡng cực của vấn đề này nếu chúng ta thêm các nút bị cô lập nk vào cả hai lớp.


Cảm ơn các liên kết đến EGRES. Tôi thực sự thích các vấn đề mở đặc biệt là những vấn đề liên quan đến (hoàn hảo) phù hợp.
Mohammad Al-Turkistany

Các trang web vấn đề mở chất lượng khác (liên quan đến độ phức tạp tính toán) là gì?
Mohammad Al-Turkistany

@turkistany, tôi không biết ai khác, tôi nghĩ đây cũng là về nghiên cứu hoạt động / lý thuyết đồ thị.
domotorp

3

Chú ý: Đây là một câu trả lời một phần dựa trên phỏng đoán và tin đồn! Trong khi vấn đề chung hơn của David Eppstein là NP-đầy đủ, có thể vấn đề này nằm ở P.

Hãy để chúng tôi nói rằng một đồ thị lưỡng cực với | Một | = | B | = =(AB,E)|A|=|B|=n

  • nó không được chứa 2 kết hợp hoàn hảo
  • trình tự mức độ A, khi sắp xếp thứ tự tăng dần, phải componentwise (1,2,...,n) , và tương tự như vậy đối với B. Tôi sẽ gọi đây là "điều kiện độ."

Cho đến nay, tôi không thể tìm thấy bất kỳ ví dụ nào mà đồ thị đáp ứng các điều kiện này, nhưng không thể là UPMX. Trong trường hợp đó, có lẽ họ là đủ. Người ta có thể chứng minh điều này bằng thuật toán sau:

  1. nếu biểu đồ có> 1 kết quả khớp hoàn hảo, hãy trả về "không phải UPMX"
  2. nếu biểu đồ không đạt điều kiện độ, trả về "không phải UPMX"
  3. nếu đồ thị có = 1 kết hợp hoàn hảo, hãy trả về "UPMX"
  4. nếu không, có lẽ chúng ta có thể chỉ ra nó là UPMX. Có lẽ thuật toán sau có thể chứng minh điều đó:
    • (n+12)2
    • tìm một số cạnh mới mà sự bổ sung của nó không tạo ra một kết hợp hoàn hảo và không vi phạm điều kiện mức độ; thêm e vào biểu đồ
  5. (n+12)1 cạnh và không khớp hoàn hảo, và thỏa mãn điều kiện độ. Tôi nghĩ không quá khó để thể hiện nó là UPMX, do đó cũng là biểu đồ ban đầu.

Bạn có thể mô tả các cạnh mới nào sẽ tạo ra một kết hợp hoàn hảo bằng cách sử dụng định lý của Hall và không khó để mô tả các cạnh mới nào sẽ vi phạm mức độ giới hạn. Thật không may, ngay cả khi đúng là một cạnh đúng loại luôn tồn tại, tôi đã không thể chứng minh điều đó.


Không phải là một cách tiếp cận xấu, tôi tự hỏi nếu nó là sự thật.
domotorp

3

Bài viết này, Lấy ma trận tam giác bằng các hoán vị cột hàng độc lập Fertin, Rusu và Vialette, cho thấy vấn đề là NP hoàn chỉnh cho ma trận vuông nhị phân.


Điều này khá đáng tiếc khi họ cũng đã chứng minh kết quả tương tự một cách độc lập với chúng tôi, tôi đoán chúng ta nên giao tiếp tốt hơn. Nhưng dù sao, tôi sẽ gửi email cho họ.
domotorp

@domotorp Vấn đề tương tự đã được hỏi trên MathOverflow và câu trả lời tốt nhất là nó nằm trong "NP-limbo". mathoverflow.net/questions/191963/...
Mohammad Al-Turkistany

-1

Vấn đề là NP-đầy đủ nhưng thuật toán để giải quyết nó ở đâu? Tôi có một thuật toán hoạt động trên nhiều ví dụ, nhưng tôi không thể chứng minh rằng nó sẽ hoạt động mọi lúc.


1
Bạn có thể mô tả một lớp biểu đồ thú vị mà thuật toán của bạn là chính xác không?
RB
Khi sử dụng trang web của chúng tôi, bạn xác nhận rằng bạn đã đọc và hiểu Chính sách cookieChính sách bảo mật của chúng tôi.
Licensed under cc by-sa 3.0 with attribution required.